Mathcenter Forum

Mathcenter Forum (https://www.mathcenter.net/forum/index.php)
-   อสมการ (https://www.mathcenter.net/forum/forumdisplay.php?f=18)
-   -   อสมการ (https://www.mathcenter.net/forum/showthread.php?t=786)

aaaa 25 กุมภาพันธ์ 2005 09:17

อสมการ
 
หายไปนานกับโจทย์อสมการ กลับมาอีกครั้งพร้อมด้วยโจทย์อสมการหลายๆแบบ

1) (BMO'99) กำหนดให้ \( p,q,r\geq0\) โดย \( p+q+r=1 \) จงพิสูจน์ว่า
\[
ึ7(pq+qr+rp)\leq2+9pqr
\]
2) (Belarus'99) กำหนดให้ \( a,b,c\geq0\) และ \( a^2+b^2+c^2=3\) จงพิสูจน์ว่า
\[
\frac{1}{1+ab}+\frac{1}{1+bc}+\frac{1}{1+ca}\geq\frac{3}{2}
\]
3) (Carlson's inequality) กำหนดให้ \(a,b,c\geq0 \) จงแสดงว่า
\[
\sqrt[3]{\frac{(a+b)(b+c)(c+a)}{8}}\geq\sqrt{\frac{ab+bc+ca}{3}}
\]
4) (French'05) กำหนดให้ \( x,y,z\geq0 \) โดย \( x^2+y^2+z^2=25 \) จงหาค่าต่ำสุดของ
\[
\frac{xy}{z}+\frac{yz}{x}+\frac{zx}{y}
\]
5) (IMO Shortlist'96) กำหนดให้ \( a,b,c>0 \) โดย \( abc=1 \) จงพิสูจน์ว่า
\[
\frac{ab}{a^5+b^5+ab}+\frac{bc}{b^5+c^5+bc}+\frac{ca}{c^5+a^5+ca}\leq1
\]

nooonuii 25 กุมภาพันธ์ 2005 09:44

ขอข้อสองก่อนละกันครับ

\[ \frac{1}{1+ab} + \frac{1}{1+bc}+\frac{1}{1+ca}\geq \frac{9}{(1+ab)+(1+bc)+(1+ca)}\geq \frac{9}{3+a^2+b^2+c^2} = \frac{3}{2} \]

nooonuii 25 กุมภาพันธ์ 2005 10:05

ข้อ 5 ครับ

\[ a^2b^2(a+b) \leq \frac{a^4+b^4}{2} (a+b) \leq a^5 + b^5 \]

\[ a^2b^2(a+b+c) = a^2b^2(a+b) + a^2b^2c =a^2b^2(a+b) + ab \leq a^5+b^5 +ab \]

\[ \frac{ab}{a^5+b^5+ab} \leq \frac{1}{ab(a+b+c)} \]

\[ \frac{ab}{a^5+b^5+ab} + \frac{bc}{b^5+c^5+bc} + \frac{ca}{c^5+a^5+ca} \leq \frac{1}{a+b+c} (\frac{1}{ab}+\frac{1}{bc}+\frac{1}{ca}) = \frac{1}{abc} = 1 \]

nooonuii 25 กุมภาพันธ์ 2005 10:18

ข้อนี้เป็นโจทย์อสมการล่าสุดที่ผมสร้างไว้ครับ ว่าจะเอามาเล่นช่วงปีใหม่แต่ไม่ได้เล่น :D

6. ให้ a,b,c >0 โดยที่ \( a^2 + b^2 + c^2 = 12 \) จงหาค่าสูงสุดของ

\[ \sqrt{a+\sqrt{b+c}} + \sqrt{b+\sqrt{c+a}} + \sqrt{c+\sqrt{a+b}} \]

aaaa 25 กุมภาพันธ์ 2005 10:36

ใช้ อสมการ AM-GM ได้ว่า
\[
\sqrt{a+\sqrt{b+c}}=\frac{1}{2}2\cdot\sqrt{a+\sqrt{b+c}}\leq\frac{1}{4} \left(2^2+a+\sqrt{b+c}\right)\leq\frac{1}{2}\left(4+a+\frac{1}{4}(2^2+b+c)\right)
\]
และเนื่องจาก
\[
a+b+c\leq\sqrt{3(a^2+b^2+c^2)}=6
\]
ดังนั้นจบ

gools 25 กุมภาพันธ์ 2005 22:34

ข้อ 1 ครับ
เนื่องจาก \(p(1-3r)(q-2)\)\((\frac{p-3r+q-1}{3})^{3}=(\frac{1-4r-1}{3})^{3}=\frac{-64r^{3}}{27}\)\(0\)
\p(1-3r)(q-2)+q(1-3p)(r-2)+r(1-3q)(p-2)0
(p-3rp)(q-2)+(q-3pq)(r-2)+(r-3qr)(p-2)0
p(q-2)+q(r-2)+r(p-2)3pq(r-2)+3qr(p-2)+3rp(q-2)
pq+qr+rp-2(p+q+r)3pqr+3pqr+3pqr-6pq-6qr-6rp
เนื่องจาก p+q+r=1 ดังนั้น
7(pq+qr+rp)2+9pqr

nooonuii 26 กุมภาพันธ์ 2005 05:33

โจทย์ข้อ 6 ที่คุณ aaaa เฉลยไว้ ผมลองดูแล้วรู้สึกว่าจะยังไม่ sharp นะครับ
ไม่รู้ว่าผมคิดผิดตรงไหนรึปล่าวเหอเหอ มันลายตาครับ คุณ aaaa ช่วยตรวจทานอีกทีครับ :D

nooonuii 26 กุมภาพันธ์ 2005 05:59

ข้อสามครับ

\( (a+b)(b+c)(c+a) = (a+b+c)(ab+bc+ca) - abc \)

\(\large{ = (a+b+c)(ab+bc+ca) - \sqrt[3]{(ab)(bc)(ca)} \sqrt[3]{abc} } \)

\( \geq (a+b+c)(ab+bc+ca) - (\frac{ab+bc+ca}{3})( \frac{a+b+c}{3}) \)

\( = \frac{8}{9} (a+b+c)(ab+bc+ca) \)

\( \geq 8 (\frac{ab+bc+ca}{3})^{3/2} \)

อสมการสุดท้ายมาจาก \( 3(ab+bc+ca) \leq (a+b+c)^2 \)

aaaa 26 กุมภาพันธ์ 2005 06:00

ขออภัยครับคุณ nooonuii เมื่อวานรีบมากไปหน่อย คือที่มาของวิธีทำคือ ผมมองว่าอสมการจะเท่ากับก็ต่อเมื่อ \( a=b=c=2\)
ซึ่งในกรณีนี้จะได้ว่า
\[
\sqrt{a+\sqrt{b+c}}=\sqrt{b+c}=2
\]
เทอมอื่นก็เช่นเดียวกัน ดังนั้นผมเลยเอาเทอม 2 คูณเข้าและหารออกและใช้ AM-GM จะได้
\[
\sqrt{a+\sqrt{b+c}}=\frac{1}{2}\left(2\cdot\sqrt{a+\sqrt{b+c}}\right)\leq\frac{1}{4}\left(2^2+(a+\sqrt{b+c})\right)
\]
ทำทำนองเดียวกับ เทอม \( \sqrt{b+c} \) จะได้ว่า
\[
\sqrt{b+c}\leq\frac{1}{4}\left(2^2+b+c\right)
\]
ดังนั้นเราจะได้ว่า
\[
\sum_{\text{sym}}\sqrt{a+\sqrt{b+c}}\leq\sum_{\text{sym}}\frac{1}{4}\left(4+a+\frac{1}{4}(4+b+c)\right)=\frac{15}{4}+\frac{3}{8} (a+b+c)
\]
ก็เหลือแสดงว่า
\[
a+b+c\leq6
\]
ซึ่งจะได้ตามมาว่า ค่ามากสุดเท่ากับ 6

nooonuii 26 กุมภาพันธ์ 2005 06:11

เข้าใจแล้วครับ ปรากฎว่า อสมการสุดท้ายของบรรทัดแรก ที่คุณ aaaa เฉลยไว้ตอนแรกมันเป็น 1/2 นี่เองครับ จริงๆมันต้อง 1/4 ผมก็เอาตรงนี้มาบวกกันมันเลยได้เป็น 12 แทน

วิธีคิดของผม ใช้เครื่องมือกันคนละแบบครับ

\[ \sqrt{a+\sqrt{b+c}} + \sqrt{b+\sqrt{c+a}} + \sqrt{c+\sqrt{a+b}} \leq \sqrt{3(a+b+c+\sqrt{a+b} + \sqrt{b+c} + \sqrt{c+a})} \leq \sqrt{3(a+b+c + \sqrt{6(a+b+c)})}\]

แล้วก็ใช้ a+b+c 6 จะได้ค่าสูงสุดเป็น 6 ครับ

nooonuii 26 กุมภาพันธ์ 2005 06:27

7. ให้ a,b,c เป็นจำนวนจริง จงพิสูจน์ว่า
\[ \large { \sqrt{a^2 + (1-b)^2} + \sqrt{b^2 + (1-c)^2} + \sqrt{c^2 + (1-a)^2} \geq \frac{3\sqrt{2}}{2} } \]

aaaa 26 กุมภาพันธ์ 2005 06:36

ขออนุญาติให้ข้อสังเกตุนะครับ

ข้อ 5) ในวิธีทำของคุณ nooonuii สวยงามมากทีเดียวครับ แต่ตอนแรกผมก็มองอสมการที่สองของบรรทัดแรกอยู่นาน เพิ่งจะมองออกว่าใช้
rearrangement ineq นั่นเองครับ วิธีผมคิดเป็นดังนี้ครับ (อยากให้เสนอแนวคิดกันหลายๆแบบครับ จะได้เป็นการแลกเปลี่ยนกัน)
พิจารณาเทอม
\[
a^5+b^5=(a+b)\left[a^4-a^3b+a^2b^2-ab^3+b^4\right]=(a+b)\left[(a-b)^2(a^2+ab+b^2)+(ab)^2\right]\geq(a+b)(ab)^2
\]
ดังนั้น
\[
\frac{ab}{a^5+b^5+ab}\leq\frac{ab}{(a+b)(ab)^2+ab}=\frac{1}{(a+b)/c+1}=\frac{c}{a+b+c}
\]
ทำนองเดียวกันสำหรับเทอมอื่นๆ ดังนั้นก็จบ

ข้อ 6) อืมผมว่าผมทำแบบ basic มากไปเลยออกมายืดยาว แบบของคุณ nooonuii สั้นกว่าเยอะเลยครับ
แต่แนวคิดก็เหมือนกัน :)

ข้อ 1) ของ gools ตรงการใช้อสมการ AM-GM ในบรรทัดแรกนี่ ผมว่าน่าจะผิดครับ เพราะอสมการ AM-GM ต้องใช้เมื่อทุกเทอมมากกว่าหรือเท่ากับศูนย์

nooonuii 26 กุมภาพันธ์ 2005 08:46

ข้อ 4 ครับ

จาก 3(ab+bc+ca) (a+b+c)2 จะได้

\[ \large{ 3[(xy)^2(yz)^2+(yz)^2(zx)^2+(zx)^2(xy)^2] \leq [(xy)^2+(yz)^2+(zx)^2]^2 } \]

\[ \large{ 3(xyz)^2[x^2+y^2+z^2] \leq [(xy)^2+(yz)^2+(zx)^2]^2} \]

\[ \large{ \frac{xy}{z} + \frac{yz}{x} + \frac{zx}{y} \geq 5\sqrt{3} } \]

nooonuii 26 กุมภาพันธ์ 2005 08:54

ข้อ 1 ของน้อง gool นี่ตอนแรกผมคิดว่าน่าจะ modify ได้ แต่ปรากฎว่าทำไม่ได้ครับ เฮ้อคงต้องรอคุณ aaaa มาเฉลยแล้วล่ะครับ เพราะข้อนี้ผมคิดมาอย่างน้อยสามรอบแล้ว โจทย์ BMO นี่ยากจริงๆครับ

nooonuii 26 กุมภาพันธ์ 2005 09:11

โจทย์ของเวียดนาม คู่แข่งที่นำหน้าเราอยู่ประมาณห้าพันลี้ครับ

8. Vietnam'96 ให้ a,b,c,d 0 โดยที่ 2(ab+bc+cd+da+ac+bd) + abc + bcd + cda + dab = 16 จงแสดงว่า
\[ 3(a+b+c+d) \geq 2(ab+bc+cd+da+ac+bd) \]


เวลาที่แสดงทั้งหมด เป็นเวลาที่ประเทศไทย (GMT +7) ขณะนี้เป็นเวลา 01:29

Powered by vBulletin® Copyright ©2000 - 2024, Jelsoft Enterprises Ltd.
Modified by Jetsada Karnpracha